2024 AMC 12A Problems/Problem 15

Revision as of 21:43, 20 March 2025 by Maa is stupid (talk | contribs)

Problem

Suppose $p$ and $q$ are real numbers for which \[\log_{p}(q) - \log_{2p}(q) =\frac{1}{3} \qquad \operatorname{and} \qquad \log_{2p}(q) - \log_{4p}(q) =\frac{1}{4}.\] What is the value of $\log_{4p}(q) - \log_{8p}(q)$?

$\textbf{(A)}~\frac{1}{6}\qquad\textbf{(B)}~\frac{7}{40}\qquad\textbf{(C)}~\frac{8}{45}\qquad\textbf{(D)}~\frac{7}{36}\qquad\textbf{(E)}~\frac{1}{5}$

Solution

Let $x = \log_{2}p$ and $y = \log_{2}q$. Then note \[\log_{p}(q) - \log_{2p}(q) =\frac{\log_{2}(q)}{\log_{2}(p)} -\frac{\log_{2}q}{\log_{2}(2p)} =\frac{y}{x} -\frac{y}{x + 1} =\frac{y}{x(x + 1)} =\frac{1}{3} \implies x(x + 1) = 3y.\] Similarly, \[\log_{2p}(q) - \log_{4p}(q) =\frac{\log_{2}(q)}{\log_{2}(2p)} -\frac{\log_{2}(q)}{\log_{2}(4p)} =\frac{y}{x + 1} -\frac{y}{x + 2} =\frac{y}{(x + 1)(x + 2)} =\frac{1}{4} \implies (x + 1)(x + 2) = 4y.\] Dividing the two equations, $\tfrac{x}{x + 2} = \tfrac{3}{4}$ which solves to $x = 6$. Substituting, $y = \tfrac{1}{3} \cdot 6 \cdot 7 = 14$ and \[\log_{4p}(q) - \log_{8p}(q) =\frac{y}{x + 2} -\frac{y}{x + 3} = \frac{14}{8} - \frac{14}{9} = \boxed{\textbf{(D)}~\frac{7}{36}}.\]

See also

2024 AMC 12A (ProblemsAnswer KeyResources)
Preceded by
Problem 14
Followed by
Problem 16
1 2 3 4 5 6 7 8 9 10 11 12 13 14 15 16 17 18 19 20 21 22 23 24 25
All AMC 12 Problems and Solutions

These problems are copyrighted © by the Mathematical Association of America, as part of the American Mathematics Competitions. AMC Logo.png